The chairperson of Acme Corporation has decided to move the company from its current location in Milltown to Ocean Vi...

Theresa on November 20, 2015

Confused

Please explain why the correct answer is E.

Reply
Create a free account to read and take part in forum discussions.

Already have an account? log in

Mehran on November 21, 2015

Thank you for your question, @Theresaturner. This is a strengthen with necessary premise question. Let's first be sure we understand the reasoning in the stimulus.

The stimulus presents an argument, as follows:

Premise: Chairperson has decided to move company to Ocean View

Premise: Most employees cannot afford housing within a 30-minute commute of Ocean View

Conclusion: So after the move, most employees will have a commute of more than 30 minutes.

Now, this conclusion does not follow logically from these premises. For example, it could be true that most of the company's current employees could choose to leave the company rather than follow it from Milltown to Ocean View, right?

On a strengthen with necessary premise question, we need to search through the answer choices to find the missing premise that is necessary for the conclusion in the stimulus to follow logically. You can test answer choices by negating them--if you negate the correct answer, the argument in the stimulus would fall apart.

Answer choice (E) is the correct answer. It states that "Acme's move to Ocean View will not be accompanied by a significant pay raise for Acme employees."

Let's negate that sentence: "Acme's move to Ocean View will (or may) be accompanied by a significant pay raise for Acme employees." Well, if that was true, then it would no longer be the case that most employees cannot afford housing within 30 minutes of Ocean View--raise means more income means ability to afford higher housing costs. Thus, if we negate answer choice (E) and add it to the stimulus, the argument in the stimulus would fall apart. That's how we know that answer choice (E) is a necessary premise to strengthen the argument in the stimulus.

Hope this helps! Please let us know if you have any additional questions.